K
Khách

Hãy nhập câu hỏi của bạn vào đây, nếu là tài khoản VIP, bạn sẽ được ưu tiên trả lời.

9 tháng 6 2016

a)

\(x^4-y^4=\left(x^2-y^2\right)\left(x^2+y^2\right)=\left(x-y\right)\left(x+y\right)\left(x^2+y^2\right)\)

\(=\left(x-y\right)\left(x^3+x^2y+xy^2+y^3\right).\)

b) 

\(\left(x+y+z\right)\left(x^2+y^2+z^2-xy-yz-zx\right)=x^3+x^2y+x^2z+xy^2+y^3+y^2z+\)

\(+xz^2+yz^2+z^3-x^2y-xy^2-xyz-xyz-y^2z-yz^2-x^2z-xyz-xz^2=\)

\(=x^3+y^3+z^3-3xyz\)

NV
3 tháng 4 2021

Cần thêm điều kiện x;y;z đôi một phân biệt và để dấu "=" xảy ra khi thì x;y;z không âm chứ không phải dương

Không mất tính tổng quát, giả sử \(z=min\left\{x;y;z\right\}\Rightarrow xy+yz+zx\ge xy\)

\(\Rightarrow\dfrac{4}{xy+yz+zx}\le\dfrac{4}{xy}\)

Đồng thời: 

\(\left(z-x\right)^2=x^2+z\left(z-2x\right)\le x^2\Rightarrow\dfrac{1}{\left(z-x\right)^2}\ge\dfrac{1}{x^2}\) 

\(\left(y-z\right)^2=y^2+z\left(z-2y\right)\le y^2\ge\dfrac{1}{\left(y-z\right)^2}\ge\dfrac{1}{y^2}\)

Nên ta chỉ cần chứng minh:

\(\dfrac{1}{\left(x-y\right)^2}+\dfrac{1}{x^2}+\dfrac{1}{y^2}\ge\dfrac{4}{xy}\)

\(\Leftrightarrow\dfrac{xy}{\left(x-y\right)^2}+\dfrac{x^2+y^2}{xy}\ge4\)

\(\Leftrightarrow\dfrac{xy}{\left(x-y\right)^2}+\dfrac{\left(x-y\right)^2}{xy}\ge2\) (hiển nhiên đúng theo AM-GM)

28 tháng 5 2021

dấu = xảy ra khi nào ạ ?

25 tháng 11 2017

Áp dụng bđt a^2+b^2+c^2 >= ab+bc+ca và a^2+b^2+c^2 >= (a+b+c)^3/3 thì :

x^4+y^4+z^4 >= x^2y^2+y^2z^2+z^2x^2 >= (xy+yz+zx)^2/3 = 4^2/3 =  16/3 ( ĐPCM )

Dấu "=" xảy ra <=> x=y=z và xy+yz+zx=4 <=> x=y=z = +-\(\frac{2}{\sqrt{3}}\)

k mk nha

27 tháng 7 2016

Ta có đẳng thức:

\(a^2+b^2+c^2\ge\frac{\left(a+b+c\right)^2}{3}\)

\(A=x^4+y^4+z^4\ge x^2y^2+y^2z^2+z^2x^2\ge\frac{\left(xy+yz+zx\right)^2}{3}=\frac{1}{3}\)

\(\Rightarrow Min_A=\frac{1}{3}\)khi \(x=y=z=\frac{1}{\sqrt{3}}\)

hoặc bạn áp dụng hệ thức holder á

27 tháng 7 2016

Ta có:

\(x^4+y^4+z^4\ge x^2y^2+y^2z^2+z^2x^2\)

Mặt khác:

\(\left(xy+yz+zx\right)^2=1\le3\left(x^2y^2+y^2z^2+z^2x^2\right)\)

\(\Rightarrow\frac{1}{3}\le\left(x^2y^2+y^2z^2+z^2x^2\right)\)

hay \(x^4+y^4+z^4\ge\frac{1}{3}\Rightarrow A\ge\frac{1}{3}\)

Vậy \(Min_A=\frac{1}{3}\)khi \(x=y=z=\frac{1}{\sqrt{3}}\)

22 tháng 7 2017

a, \(x^3+y^3+z^3=3xyz\Rightarrow x^3+y^3+z^3-3xyz=0\)( 1 )

Nhận xét  :   \(\left(x+y\right)^3=x^3+y^3+3x^2y+3xy^2\Rightarrow x^3+y^3=\left(x+y\right)^3-3x^2-3xy^2\)

Thay vào ( 1 ) ta có  :  

\(\left(x+y\right)^3+c^3-3x^2y-3xy^2-3xyz\)

\(=\left(z+y+z\right)\left[\left(x+y\right)^2-\left(x+y\right)z+z^2\right]-3xy\left(x+y+z\right)\)

\(=\left(z+y+z\right)\left(z^2+2xy+y^2-xz-yz+z^2\right)-3xyz\left(z+y+z\right)\)

\(=\left(x+y+z\right)\left(x^2+2xy+y^2-xz-yz+z^2-3xy\right)\)

\(=\left(x+y+z\right)\left(z^2+x^2+y^2-xy-yz-xz\right)\)

Vì theo đầu bài ta có: \(x+y+z=0\)nên ta có ( DPCM ) ..... học cho tốt nhé!